In diesem Kapitel führen wir einen Satz ein, der eine hinreichende Bedingung gibt, unter der die Umkehrabbildung einer bijektiven Funktion wieder stetig ist. In der Literatur wird der Satz manchmal Satz von der Stetigkeit der Umkehrabbildung oder Umkehrsatz genannt. Das erstaunliche an diesem Resultat ist, dass die Umkehrfunktion einer unstetigen Funktion sehr wohl stetig sein kann.

Motivation

error: non-centered image not implemented, yet!
Wir wollen uns eine möglichst allgemeine Bedingung überlegen, wann eine bijektive Funktion f:D\to W mit D,W\subseteq \mathbb{R} eine stetige Umkehrfunktion besitzt. Der erste Ansatzpunkt, den wir dabei natürlicherweise untersuchen, ist die Stetigkeit von f. Spontan würden wir vermuten, dass aus der Stetigkeit von f auch die von f^{{-1}} folgt. Das dem nicht so ist, zeigt folgendes Beispiel:
{\begin{aligned}f&:([-1,0]\cup (1,2])\to [-1,1]\\[1em]&:x\mapsto f(x)={\begin{cases}x&-1\leq x\leq 0\\x-1&1<x\leq 2\end{cases}}\end{aligned}}
error: non-centered image not implemented, yet!
Die Funktion ist stetig, da sie sowohl auf [-1,0] als auch auf (1,2] stetig ist. Außerdem ist sie streng monoton steigend, und damit injektiv. Das Bild von f ist [-1,1] und damit ist f:([-1,0]\cup (1,2])\to [-1,1] surjektiv. Insgesamt ist f bijektiv und damit umkehrbar. Die Umkehrabbildung f^{{-1}}:[-1,1]\to ([-1,0]\cup (1,2]) besitzt die Funktionsvorschrift:
f^{{-1}}(y)={\begin{cases}y&-1\leq y\leq 0\\y+1&0<y\leq 1\end{cases}}
Diese Umkehrfunktion ist nicht stetig, da sie bei y=0 eine Sprungstelle besitzt. Es kann also tatsächlich vorkommen, dass eine stetige Funktion eine unstetige Umkehrfunktion besitzt.
Auch eine andere Sache zeigt sich: Die Umkehrfunktion f^{{-1}}=g ist ebenfalls umkehrbar mit Umkehrfunktion g^{{-1}}=\left(f^{{-1}}\right)^{{-1}}=f. Dies bedeutet aber, dass eine unstetige Funktion (wie g) eine stetige Umkehrfunktion haben kann. Wir halten daher fest:
Die Stetigkeit / Unstetigkeit einer Funktion hat keinerlei Einfluss auf die Stetigkeit / Unstetigkeit ihrer Umkehrfunktion.
Das Problem liegt im Definitionsbereich von f. Dieser ist [-1,0]\cup (1,2], ist also kein Intervall. Der Definitionsbereich ist nicht zusammenhängend und besitzt eine „Lücke“. Wie wirkt sich diese „Lücke“ nun auf die Umkehrfunktion f^{{-1}} aus?
Da der Definitionsbereich von f der Zielmenge von f^{{-1}} entspricht, hat somit die Umkehrfunktion diese „Lücke“ in der Zielmenge. Gleichzeitig wird aus der Zielmenge von f der Definitionsbereich von f^{{-1}}. Damit kann aber f^{{-1}} nicht stetig sein. Der Definitionsbereich von f^{{-1}} ist ein Intervall und damit zusammenhängend, während die Zielmenge eine Lücke besitzt. Da f^{{-1}} surjektiv ist und jede Zahl aus dem Zielbereich annehmen muss, muss der Graph von f^{{-1}} einen Sprung aufweisen. Somit ist f^{{-1}} unstetig.
Wir müssen daher als Voraussetzung fordern, dass der Definitionsbereich von f ein Intervall ist, um diese Problematik zu verhindern. In der Tat reicht diese Forderung aus, damit f^{{-1}} stetig ist. Dies werden wir mit Hilfe der \epsilon -\delta -Charakterisierung der Stetigkeit beweisen. Mit Hilfe des error: internal links not implemented, yet! können wir außerdem folgern, dass der Zielbereich von f und damit der Definitionsbereich von f^{{-1}} ein Intervall ist.

Satz von der Stetigkeit der Umkehrfunktion error: TODO

Satz: Stetigkeit der Umkehrfunktion
Sei D\subseteq \mathbb{R} ein Intervall und f:D\to W eine surjektive, streng monotone und stetige Abbildung. Dann ist f bijektiv, und die Umkehrfunktion
f^{{-1}}:W\to D
ist stetig und im gleichen Sinn wie f streng monoton. Außerdem ist f(D)=W ein Intervall.
Beweis
Beweisschritt: f ist bijektiv
Seien x,y\in D mit x\neq y. Also ist x<y oder x>y. Da f streng monoton ist, folgt daraus f(x)<f(y) oder f(x)>f(y). In jedem Fall gilt f(x)\neq f(y). Also ist f injektiv. Da f als surjektiv vorausgesetzt ist, ist die Abbildung bijektiv.
Beweisschritt: f^{{-1}} ist streng monoton
Sei f ohne Beschränkung der Allgemeinheit streng monoton steigend, d.h. aus x,y\in D mit x<y folgt f(x)<f(y). Sei nun {\tilde x},{\tilde y}\in W mit {\tilde x}<{\tilde y}. Wir müssen nun zeigen, dass f^{{-1}}({\tilde x})<f^{{-1}}({\tilde y}) ist.
Nun gibt es x,y\in D mit f(x)={\tilde x} und f(y)={\tilde y}. Damit ist f^{{-1}}({\tilde x})=f^{{-1}}(f(x))=x und f^{{-1}}({\tilde y})=f^{{-1}}(f(y))=y. Da f^{{-1}} injektiv ist, ist x\neq y. Auch kann nicht x>y sein. Sonst müsste {\tilde x}=f(x)>f(y)={\tilde y} sein, was wegen {\tilde x}<{\tilde y} ausgeschlossen ist.
Also ist x<y bzw. f^{{-1}}({\tilde x})<f^{{-1}}({\tilde y}). Dies beweist, dass f^{{-1}} wie f streng monoton steigt. Im Fall, dass f streng monoton fällt, kann analog bewiesen werden, dass auch f^{{-1}} streng monoton fällt.
Beweisschritt: f^{{-1}} ist stetig
Wir verwenden die \epsilon -\delta -Charakterisierung der Stetigkeit. Sei hierzu \epsilon >0 und b\in W=f(D) beliebig. Sei weiter a=f^{{-1}}(b) das Urbild von b unter f, d.h. f(a)=b. Wir müssen nun zeigen, dass es ein \delta >0 gibt, so dass |f^{{-1}}(y)-f^{{-1}}(b)|<\epsilon für alle y\in W mit |y-b|<\delta ist. Damit ist f^{{-1}} stetig in b. Da b\in W beliebig gewählt wurde, folgt daraus die Stetigkeit von f^{{-1}}:W\to D. Wir führen nun eine Fallunterscheidung darin durch, ob a ein Randpunkt von D ist:
Fall 1:
a ist kein Randpunkt von D
Da a kein Randpunkt von D ist, gibt es ein {\tilde \epsilon } mit 0<{\tilde \epsilon }<\epsilon und ]a-{\tilde \epsilon },a+{\tilde \epsilon }[\subseteq D. Wir setzen b_{1}=f(a-{\tilde \epsilon }) und b_{2}=f(a+{\tilde \epsilon }). Ist f streng monoton steigend, so gilt b_{1}<b<b_{2}. Ist f streng monoton fallend, so ist b_{2}<b<b_{1}. Damit bildet f das Intervall [a-{\tilde \epsilon },a+{\tilde \epsilon }] bijektiv auf [b_{1},b_{2}] bzw. [b_{2},b_{1}] ab. Setzen wir
\delta =\min\{b-b_{1},b-b_{2}\}
so folgt
f^{{-1}}(]b-\delta ,b+\delta [)\subseteq ]a-{\tilde \epsilon },a+{\tilde \epsilon }[\subseteq ]a-\epsilon ,a+\epsilon [
Damit ist |f^{{-1}}(y)-f^{{-1}}(b)|<\epsilon für alle y\in W mit |y-b|<\delta erfüllt. Also ist f^{{-1}} stetig in b.
Fall 2:
a ist ein Randpunkt von D
Nehmen wir ohne Beschränkung der Allgemeinheit an, dass a das Maximum von D ist. Nun gibt es ein {\tilde \epsilon } mit 0<{\tilde \epsilon }<\epsilon , so dass a-{\tilde \epsilon } in D liegt. Damit existiert f(a-{\tilde \epsilon }) und wir können \delta =|f(a)-f(a-{\tilde \epsilon })| setzen.
Wenn f streng monoton steigt, dann bildet f das Intervall [a-{\tilde \epsilon },a] bijektiv auf [f(a-{\tilde \epsilon }),f(a)]=[b-\delta ,b] ab. Dann ist f^{{-1}}([b-\delta ,b])=[a-{\tilde \epsilon },a]\subseteq (a-\epsilon ,a+\epsilon ). Weil es kein y\in W mit y>b gibt, gilt somit |f^{{-1}}(y)-f^{{-1}}(b)|<\epsilon für alle y\in W mit |y-b|<\delta . Dies beweist die Stetigkeit von f^{{-1}} in b. Analog kann vorgegangen werden, wenn f monoton fällt oder a das Minimum von D ist.

Beispiele

Wurzelfunktionen

Beispiel: Wurzelfunktionen
Beispiel
Ist k\in \mathbb{N} mit k\geq 2. Dann ist die k-te Potenzfunktion
f:\mathbb{R} ^{+}\to \mathbb{R} ^{+}:x\mapsto f(x)=x^{k}
streng monoton steigend, denn für x,y\in \mathbb{R} ^{+} mit x<y gilt
f(x)=x^{k}=\underbrace {x\cdot \ldots \cdot x}_{{k{\text{-mal}}}}<\underbrace {y\cdot \ldots \cdot y}_{{k{\text{-mal}}}}=y^{k}=f(y)
Also ist f injektiv. Weiter ist f surjektiv, denn zu jedem y\in \mathbb{R} ^{+} gilt für x={\sqrt[ {k}]{y}}:
f(x)=f\left({\sqrt[ {k}]{y}}\right)=\left({\sqrt[ {k}]{y}}\right)^{k}=y
Die Umkehrfunktion ist die k-te Wurzelfunktion f^{{-1}}:\mathbb{R} ^{+}\to \mathbb{R} ^{+} mit f(x)={\sqrt[ {k}]{x}}. Nach dem Satz zur Stetigkeit der Umkehrfunktion ist sie ebenfalls streng monoton wachsend und stetig.

Natürliche Logarithmusfunktionen

Beispiel: Natürliche Logarithmusfunktionen
Beispiel
Die Exponentialfunktion
f:\mathbb{R} \to \mathbb{R} ^{+},\ f(x)=\exp(x)
ist streng monoton steigend. Seien x,y\in \mathbb{R} ^{+} mit x<y. Dann gibt es ein t\in \mathbb{R} ^{+} mit y=x+t. Damit gilt
\exp(y)=\exp(x+t){\underset {{\text{gleichung}}}{{\overset {{\text{Funktional-}}}{=}}}}\exp(x)\cdot \underbrace {\exp(t)}_{{>1}}>\exp(x)
Aus der strengen Monotonie folgt, dass \exp injektiv ist. Weiter ist die Exponentialfunktion surjektiv, denn es gilt:
{\begin{aligned}\lim _{{x\to \infty }}\exp(x)&=\infty \lim _{{x\to -\infty }}\exp(x)&=0\end{aligned}}
Die Surjektivität folgt damit aus der Stetigkeit der Exponentialfunktion und dem Zwischenwertsatz. Die Umkehrfunktion ist die natürliche Logarithmusfunktion:
f^{{-1}}:\mathbb{R} ^{+}\to \mathbb{R} ,\ f(x)=\ln x
Sie ist wegen der strengen Monotonie der Exponentialfunktion auch streng monoton steigend. Aus dem Umkehrsatz der Stetigkeit folgt außerdem, dass die Logarithmusfunktion stetig ist.

Übungsaufgaben

Übungsaufgabe 1

Stetigkeit der Umkehrfunktion 1
Sei f:\mathbb{R} \to \mathbb{R} definiert durch die Zuordnungsvorschrift:
f(x)={\frac {x}{1+|x|}}
Beweise folgende Aussagen:
Übung 1:
Zeige, dass f auf \mathbb{R} stetig, streng monoton wachsend und injektiv ist.
Übung 2:
Zeige, dass f:\mathbb{R} \to (-1,1) surjektiv ist.
Übung 3:
Begründe, warum die Umkehrfunktion f^{{-1}}:(-1,1)\to \mathbb{R} existiert und bestimme die Zuordnungsvorschrift von f^{{-1}} explizit. Zeige, dass f^{{-1}} stetig und streng monoton wachsend ist.
Lösung: Lösung von Teilaufgabe 1:
f ist stetig als Quotient der stetigen Funktionen a:\mathbb{R} \to \mathbb{R} :x\mapsto x und b:\mathbb{R} \to \mathbb{R} :x\mapsto 1+|x|. Dabei ist 1+|x|\neq 0 für alle x\in \mathbb{R} . Seien x,y\in \mathbb{R} mit x<y. Zum Beweis der strengen Monotonie zeigen wir, dass f(x)<f(y) ist:
Fall 1:
x,y\geq 0
Dann gilt
x<y\iff \underbrace {x+xy}_{{=x(1+y)}}<\underbrace {y+xy}_{{=y(1+x)}}\iff \underbrace {{\frac {x}{1+x}}}_{{=f(x)}}<\underbrace {{\frac {y}{1+y}}}_{{=f(y)}}
Fall 2:
x\leq 0<y
Hier ist
f(x)=\underbrace {{\frac {x}{1+|x|}}}_{{\leq 0}}<\underbrace {{\frac {y}{1+|y|}}}_{{>0}}=f(y)
Fall 3:
x<0\leq y
Hier ist
f(x)=\underbrace {{\frac {x}{1+|x|}}}_{{<0}}<\underbrace {{\frac {y}{1+|y|}}}_{{\geq 0}}=f(y)
Fall 4:
x,y<0
Es gilt
x<y\iff \underbrace {x-xy}_{{=x(1-y)}}<\underbrace {y-xy}_{{=y(1-x)}}\iff \underbrace {{\frac {x}{1-x}}}_{{=f(x)}}<\underbrace {{\frac {y}{1-y}}}_{{=f(y)}}
Also ist f streng monoton steigend auf \mathbb{R} und damit auch injektiv.
Lösung von Teilaufgabe 2:
Es gilt für alle x\in \mathbb{R} :
|f(x)|={\frac {|x|}{1+|x|}}<{\frac {|x|}{|x|}}=1
Damit ist stets -1<f(x)<1. Außerdem ist
{\begin{aligned}\lim _{{x\to \infty }}f(x)&=\lim _{{x\to \infty }}{\frac {x}{1+|x|}}\\[0.3em]&=\lim _{{x\to \infty \atop x>0}}{\frac {x}{1+|x|}}\\[0.3em]&=\lim _{{x\to \infty \atop x>0}}{\frac {x}{1+x}}\\[0.3em]&=\lim _{{x\to \infty \atop x>0}}{\frac {1}{{\frac 1x}+1}}=1\end{aligned}}
und
{\begin{aligned}\lim _{{x\to -\infty }}f(x)&=\lim _{{x\to -\infty }}{\frac {x}{1+|x|}}\\[0.3em]&=\lim _{{x\to -\infty \atop x<0}}{\frac {x}{1+|x|}}\\[0.3em]&=\lim _{{x\to -\infty \atop x<0}}{\frac {x}{1-x}}\\[0.3em]&=\lim _{{x\to -\infty \atop x<0}}{\frac {1}{{\frac 1x}-1}}=-1\end{aligned}}
Da f stetig ist, gibt es nach dem Zwischenwertsatz zu jedem y\in (-1,1) ein x\in \mathbb{R} mit f(x)=y. Also gilt für das Bild f(\mathbb{R} )=(-1,1), womit f:\mathbb{R} \to (-1,1) surjektiv ist.
Lösung von Teilaufgabe 3:
Da f bijektiv ist, existiert die Umkehrfunktion f^{{-1}}:(-1,1)\to \mathbb{R} . Als Umkehrfunktion ist f^{{-1}} ebenfalls bijektiv. Nach dem Satz über die Stetigkeit der Umkehrabbildung ist f^{{-1}} außerdem stetig und streng monoton steigend. Zur Berechnung von f^{{-1}} unterscheiden wir zunächst zwei Fälle:
Fall 1:
x\geq 0
{\begin{aligned}&y=f(x)={\frac {x}{1+x}}\\\iff {}&y+xy=x\\\iff {}&y=x-xy=x(1-y)\\\iff {}&x=\underbrace {{\frac {y}{1-y}}}_{{=f^{{-1}}(y)}}\end{aligned}}
Fall 2:
x<0
{\begin{aligned}&y=f(x)={\frac {x}{1-x}}\\\iff {}&y-xy=x\\\iff {}&y=x+xy=x(1+y)\\\iff {}&x=\underbrace {{\frac {y}{1+y}}}_{{=f^{{-1}}(y)}}\end{aligned}}
Ist nun x\geq 0, so ist f(x)=y\geq 0 und damit 1-y=1-|y|. Ist umgekehrt x<0, so ist auch f(x)=y<0 und damit 1+y=1-|y|. Insgesamt folgt daraus
f^{{-1}}:(-1,1)\to \mathbb{R} ,\ f^{{-1}}(y)={\frac {y}{1-|y|}}

Übungsaufgabe 2

Stetigkeit der Umkehrfunktion 2
Sei g:[0,\infty )\to \mathbb{R} mit g(x)=\exp(x)-\sin(\exp(-x))
Übung 1:
Zeige, dass g injektiv ist.
Übung 2:
Bestimme das Bild g([0,\infty )).
Übung 3:
Begründe, warum die Umkehrfunktion g^{{-1}}:g([0,\infty ))\to [0,\infty ) stetig ist.
Lösung: Lösung von Teilaufgabe 1:
g ist stetig als Komposition der stetigen Funktionen a:[0,\infty )\to \mathbb{R} :x\mapsto -x, b:[0,\infty )\to \mathbb{R} :x\mapsto \exp(x) und c:[0,\infty )\to \mathbb{R} :x\mapsto \sin(x). Weiter gilt für x,y\in [0,\infty ) mit x<y:
x<y\implies -x>-y\implies \exp(-x)>\exp(-y)
Nun ist \exp(-{\tilde x})\in (0,1] für {\tilde x}\in [0,\infty ). Da die Sinusfunktion auf dem halboffenen Intervall (0,1] streng monoton steigt, folgt weiter
\exp(-x)>\exp(-y)\implies \sin(\exp(-x))>\sin(\exp(-y))\implies -\sin(\exp(-x))<-\sin(\exp(-y))
Weil die Exponentialfunktion streng monoton wächst, gilt außerdem \exp(x)<\exp(y). Damit ist:
\underbrace {\exp(x)-\sin(\exp(-x))}_{{=g(x)}}<\underbrace {\exp(y)-\sin(\exp(-y))}_{{=g(y)}}
Also steigt g streng monoton und ist damit injektiv.
Lösung von Teilaufgabe 2:

Zunächst ist
g(0)=\exp(0)-\sin(\exp(0))=1-\sin(1)
Weiter gilt
{\begin{aligned}\lim _{{x\to \infty }}\exp(x)&=\infty \\\lim _{{x\to \infty }}\exp(-x)&=0\end{aligned}}
und daraus folgt
\lim _{{x\to \infty }}g(x)=\lim _{{x\to \infty }}[\underbrace {\exp(x)}_{{\to \infty }}-\underbrace {\sin(\exp(-x))}_{{\to \sin(0)=0}}]=\infty
Da g stetig und [0,\infty ) ein Intervall ist, ist g([0,\infty )) ebenfalls ein Intervall (siehe error: internal links not implemented, yet! ). Da g streng monoton steigt und \lim _{{x\to \infty }}g(x)=\infty ist, folgt
g([0,\infty ))=[g(0),\infty )=[1-\sin(1),\infty )
|teilaufgabe3-lösung= Da D=[0,\infty ) ein Intervall und g:[0,\infty )\to [1-\sin(1),\infty ) bijektiv ist, folgt aus dem Satz von der Stetigkeit der Umkehrfunktion, dass die Umkehrfunktion g^{{-1}}:[1-\sin(1),\infty )\to [0,\infty ) stetig ist.